1 Introduction

Let \(\mathbb {F}_q\), \(q=p^h\), be the finite field with q elements. Denote by \({\mathbb {P}}\) a finite classical polar space, i.e. the set of absolute subspaces of either a polarity or a non-singular quadratic form of a projective space \(\textrm{PG}(m,q)\). The maximal dimensional projective subspaces contained in \({\mathbb {P}}\) are the generators of \({\mathbb {P}}\).

Definition 1

Let \({\mathbb {P}}\) be a finite classical polar space of \(\textrm{PG}(m,q)\). An ovoid of \({\mathbb {P}}\) is a set of points of \({\mathbb {P}}\) that has exactly one point in common with each generator of \({\mathbb {P}}.\)

Regarding ovoids of the parabolic quadric Q(mq) of \(\textrm{PG}(m,q)\), m even, it is known that there are no ovoids if \(m>6\) (see [9, 16]). Hence for parabolic quadrics the only open problems concern \(m=4\) and \(m=6\).

In this paper we will focus on ovoids of Q(6, q). Denote by \((X_0,X_1,X_2,X_3,X_4,X_5,X_6)\) the homogenous coordinates of \(\textrm{PG}(6,q)\) and by \(H_\infty \) the hyperplane at infinity \(X_0=0\).

Up to equivalence, in \(\textrm{PG}(6,q)\) there is a unique non-degenerate quadric, the parabolic quadric Q(6, q) and in the sequel we fix the following parabolic quadric:

$$\begin{aligned} \textrm{Q}: X_0X_6+X_1X_5+X_2X_4+X_3^2=0.\end{aligned}$$

It is well known that there are no ovoids of Q(6, q) for q even [16], and thus from now on, we will consider only \(p>2\). By Definition 1, an ovoid of \(\textrm{Q}\) is a set of \(q^3+1\) pairwise non-collinear points on the quadric. We can always assume that an ovoid \(\textrm{O}_6\) of Q(6, q) contains the points (1, 0, 0, 0, 0, 0, 0) and (0, 0, 0, 0, 0, 0, 1) and hence it can be written in the following form:

$$\begin{aligned} \textrm{O}_6(f_1,f_2)= & {} \{(1,x,y,z,f_1(x,y,z),f_2(x,y,z),-z^2-yf_1(x,y,z)-xf_2(x,y,z))\}_{x,y,z\in \mathbb {F}_q}\\{} & {} \quad \cup \{(0,0,0,0,0,0,1)\}, \end{aligned}$$

for some functions \(f_i:\mathbb {F}_q^3 \longrightarrow \mathbb {F}_q\) with \(f_i(0,0,0) =0\) for \(i\in \{1,2\}\).

The set \(\textrm{O}_6(f_1,f_2)\) is an ovoid if and only if

$$\begin{aligned} \langle{} & {} (1,x_1,y_1,z_1,f_1(x_1,y_1,z_1),f_2(x_1,y_1,z_1), -z_1^2-y_1f_1(x_1,y_1,z_1)-x_1f_2(x_1,y_1,z_1)), \nonumber \\{} & {} (1,x_2,y_2,z_2,f_1(x_2,y_2,z_2),f_2(x_2,y_2,z_2), -z_2^2-y_2f_1(x_2,y_2,z_2)-x_2f_2(x_2,y_2,z_2))\rangle \ne 0 , \end{aligned}$$
(1)

for every \((x_1,y_1,z_1) \ne (x_2,y_2,z_2)\) in \({\mathbb {F}}_q^3\), where \(\langle \cdot ,\cdot \rangle \) is the symmetric form associated to the quadratic form of the quadric \(\textrm{Q}\).

The only known ovoids of \(\textrm{Q}\) are the two classes of ovoids listed in the following table.

Table 1 Known ovoids of Q(6, q)

Note that Condition (1) reads

$$\begin{aligned}{} & {} (z_1-z_2)^2 + (x_2-x_1)(f_2(x_2,y_2,z_2)-f_2(x_1,y_1,z_1))\nonumber \\{} & {} \quad +(y_2-y_1)(f_1(x_2,y_2,z_2)-f_1(x_1,y_1,z_1)) \ne 0,\end{aligned}$$
(2)

for every \((x_1,y_1,z_1) \ne (x_2,y_2,z_2)\) in \({\mathbb {F}}_q^3\).

In order to obtain non-existence results and partial classifications for the ovoids \(\textrm{O}_6(f_1,f_2)\) of \(\textrm{Q}\), we will consider the hypersurface \(\mathcal {W}_{f_1,f_2}\subset \textrm{PG}(6,q)\) defined by \(F_{f_1,f_2}(X_0,X_1,X_2,X_3,X_4,X_5,X_6)=0\), where \(F_{f_1,f_2}(1,X_1,X_2,X_3,X_4,X_5,X_6)\) equals

$$\begin{aligned} (X_3-X_6)^2 + (X_4-X_1)(f_2(X_4,X_5,X_6)-f_2(X_1,X_2,X_3))\nonumber \\ +(X_5-X_2)(f_1(X_4,X_5,X_6)-f_1(X_1,X_2,X_3)). \end{aligned}$$
(3)

Lemma 2

If \(f_1,f_2\not \equiv 0\), then \(\textrm{O}_6(f_1,f_2)\) is an ovoid of \(\textrm{Q}\) if and only if \(\mathcal {W}_{f_1,f_2}\) contains no affine \({\mathbb {F}}_q\)-rational points off the solid \(X_1-X_4=X_2-X_5=X_3-X_6=0\).

The main result of this paper is the following.

MainTheorem

Let \(f_1(X,Y,Z)=\sum _{ijk}a_{i,j,k}X^iY^jZ^k\) and \(f_2(X,Y,Z)=\sum _{ijk}b_{i,j,k}X^iY^jZ^k\). Suppose that \(q>6.3 (d+1)^{13/3}\), where \(\max \{\deg (f_1),\deg (f_2)\}=d\). If \(\textrm{O}_6(f_1,f_2)\) is an ovoid of \(\textrm{Q}\) then

$$\begin{aligned} q=3^h, \quad f_1(X,Y,Z)=X^2Y-nY^3-XZ, \quad f_2(X,Y,Z)=-1/n X^3+XY^2+YZ. \end{aligned}$$

Hence \(O_6(f_1,f_2)\) is a Thas-Kantor ovoid.

It is worth pointing out that the degree of the functions \(f_1(X,Y,Z)\) and \(f_2(X,Y,Z)\) associated with the Ree-Tits ovoid does not satisfy the bound \(q>6.3 (d+1)^{13/3}\) in our Main Theorem.

2 Classification of low degree ovoids

The aim of this section is to provide a partial classification of the ovoids of \(\textrm{Q}\) via the investigation of specific algebraic varieties; see [2] for a survey on links between algebraic varieties over finite fields and relevant combinatorial objects.

An algebraic hypersurface \(\mathcal {S}\) is an algebraic variety that may be defined by a single implicit equation. An algebraic hypersurface defined over a field \({\mathbb {K}}\) is absolutely irreducible if the associated polynomial is irreducible over every algebraic extension of \({\mathbb {K}}\). An absolutely irreducible \({\mathbb {K}}\)-rational component of a hypersurface \(\mathcal {S}\), defined by the polynomial F, is simply an absolutely irreducible hypersurface such that the associated polynomial has coefficients in \({\mathbb {K}}\) and it is a factor of F. For a deeper introduction to algebraic varieties we refer the interested reader to [10].

A fundamental tool to determine the existence of rational points in an algebraic variety over finite fields is the following result by Lang and Weil [12] in 1954, which can be seen as a generalization of the Hasse–Weil bound.

Theorem 3

(Lang–Weil Theorem) Let \(\mathcal {V}\subset {\mathbb {P}}^N({\mathbb {F}}_q)\) be an absolutely irreducible variety of dimension n and degree d. Then there exists a constant C depending only on N, n, and d such that

$$\begin{aligned} \left| \#\mathcal {V}({\mathbb {F}}_q)-\sum _{i=0}^{n} q^i\right| \le (d-1)(d-2)q^{n-1/2}+Cq^{n-1}, \end{aligned}$$
(4)

where \(\#\mathcal {V}({\mathbb {F}}_q)\) denotes the number of \({\mathbb {F}}_q\)-rational points of \(\mathcal {V}\).

Although the constant C was not computed in [12], explicit estimates have been provided for instance in [4,5,6,7,8, 13, 15] and they have the general shape \(C=f(d)\) provided that \(q>g(n,d)\), where f and g are polynomials of (usually) small degree. We refer to [5] for a survey on these bounds. Excellent surveys on Hasse–Weil and Lang–Weil type theorems are [7, 17]. We include here the following result by Cafure and Matera [5].

Theorem 4

[5, Theorem 7.1] Let \(\mathcal {V}\subset \textrm{AG}(n,q)\) be an absolutely irreducible \({\mathbb {F}}_q\)-variety of dimension \(r>0\) and degree \(\delta \). If \(q>2(r+1)\delta ^2\), then the following estimate holds:

$$\begin{aligned} |\#(\mathcal {V}\cap \textrm{AG}(n,q))-q^r|\le (\delta -1)(\delta -2)q^{r-1/2}+5\delta ^{13/3} q^{r-1}. \end{aligned}$$

As already mentioned, we will consider the hypersurface \(\mathcal {W}_{f_1,f_2}\subset \textrm{PG}(6,q)\) defined by

$$\begin{aligned} F_{f_1,f_2}(X_0,X_1,X_2,X_3,X_4,X_5,X_6)=0, \end{aligned}$$

where \(F_{f_1,f_2}(1,X_1,X_2,X_3,X_4,X_5,X_6)\) equals

$$\begin{aligned}{} & {} (X_3-X_6)^2 + (X_4-X_1)(f_2(X_4,X_5,X_6)-f_2(X_1,X_2,X_3))\\{} & {} \quad +(X_5-X_2)(f_1(X_4,X_5,X_6)-f_1(X_1,X_2,X_3)). \end{aligned}$$

For our purposes, the existence of an absolutely irreducible \({\mathbb {F}}_q\)-rational component in \(\mathcal {S}_f\) is enough to provide asymptotic non-existence results for ovoids of \(\textrm{Q}\).

Theorem 5

Let \(\mathcal {W}_{f_1,f_2}: F_{f_1,f_2}(X_0,X_1,X_2,X_3,X_4,X_5,X_6)=0\), where \(F_{f_1,f_2}\) is defined as in (3). Suppose that \(q>6.3 (d+1)^{13/3}\), \(\max \{\deg (f_1),\deg (f_2)\}=d\), and \(\mathcal {W}_{f_1,f_2}\) contains an absolutely irreducible component \(\mathcal {V}\) defined over \({\mathbb {F}}_q\). Then \(\textrm{O}_6(f_1,f_2)\) is not an ovoid of \(\textrm{Q}\).

Proof

Since \(q>6.3 (d+1)^{13/3}\), by Theorem 4, with \(\delta =d+1\) and \(r=5\), it is readily seen that \(\mathcal {V}\) contains more than \(q^3\) affine \({\mathbb {F}}_q\)-rational points. This yields the existence of at least one affine \({\mathbb {F}}_q\)-rational point \((a_1,b_1,c_1,a_2,b_2,c_2)\) in \(\mathcal {V}\subset \mathcal {W}_{f_1,f_2}\) (defined as in (3)) such that \(a_1\ne a_2\) or \(b_1\ne b_2\) or \(c_1\ne c_2\) and therefore \(\textrm{O}_6(f_1,f_2)\) is not an ovoid of \(\textrm{Q}\). \(\square \)

We also include, for seek of completeness, the following result which will be useful in the sequel.

Lemma 6

[1, Lemma 2.1] Let \(\mathcal {H},\mathcal {S}\subset \textrm{PG}(n,q)\) be two projective hypersurfaces. If \(\mathcal {H} \cap \mathcal {S}\) has a reduced absolutely irreducible component defined over \({\mathbb {F}}_q\) then \(\mathcal {S}\) has an absolutely irreducible component defined over \({\mathbb {F}}_q\).

2.1 Ovoids in Q(4, q)

A starting point in our investigation is the main result of [3] which characterizes ovoids of Q(4, q) of “small degree". Similarly to what happens in the dimension-6 case, any ovoid \(\textrm{O}_4\) of Q(4, q) can be written in the following form:

$$\begin{aligned} \textrm{O}_4(f) = \{(1,x,y,f(x,y),-y^2-xf(x,y))\}_{x,y\in \mathbb {F}_q} \cup \{(0,0,0,0,1)\}, \end{aligned}$$

for some function \(f:\mathbb {F}_q^2 \longrightarrow \mathbb {F}_q\) with \(f(0,0) =0\) and \(\textrm{O}_4(f)\) is an ovoid if and only if

$$\begin{aligned} \langle (1,x_1,y_1,f(x_1,y_1),-y_1^2-x_1f(x_1,y_1)), (1,x_2,y_2,f(x_2,y_2),-y_2^2-x_2f(x_2,y_2))\rangle \ne 0, \end{aligned}$$

for every \((x_1,y_1) \ne (x_2,y_2)\) in \({\mathbb {F}}_q^2\), where \(\langle \cdot ,\cdot \rangle \) is the symmetric for q odd (or alternating for q even) form associated to the quadratic form of the quadric Q(4, q).

Theorem 7

[3, Main Theorem] Let \(f(X,Y)=\sum _{ij}a_{i,j}X^iY^j\), with \(a_{0,1}=0\) if \(p>2\). Suppose that \(q>6.3 (d+1)^{13/3}\), where \(\deg (f)=d\). If \(\textrm{O}_4(f)\) is an ovoid of Q(4, q) then one of the following holds:

  1. 1.

    \(p=2\) and \(f(X,Y)=a_{1,0}X+a_{0,1}Y\). Hence \(O_4(f)\) is an elliptic quadric.

  2. 2.

    \(p>2\) and \(f(X,Y)=a_{p^j,0}X^{p^j}\), \(j\ge 0\). Hence \(O_4(f)\) is either an elliptic quadric or a Kantor ovoid.

Corollary 8

Let \(f(X,Y)=\sum _{ij}a_{i,j}X^iY^j\), with \(p>2\). Suppose that \(q>6.3(d+1)^{13/3}\), where \(d=\deg (f)\). If

$$\begin{aligned} f(X,Y)\ne a_{p^j,0}X^{p^j}+\frac{a_{0,1}^2}{4}X+a_{0,1}Y, \end{aligned}$$

for some \(j\ge 0\), then \(\mathcal {S}_f\) contains an absolutely irreducible component defined over \({\mathbb {F}}_q\), and so \(O_4(f)\) is not an ovoid of Q(4, q).

Proof

Let \(\mathcal {S}_f: F(X_1,X_2,X_3,X_4)=0\), where

$$\begin{aligned} F(X_1,X_2,X_3,X_4)= (X_2 -X_4)^2 + (X_1-X_3) \Big ( f(X_1,X_2)- f(X_3,X_4)\Big ), \end{aligned}$$

be the surface associated with an ovoid in Q(4, q); see [3]. As already observed in [3], up to a change of variables we can always suppose that \(a_{0,1}=0\). Suppose that this is not the case, then consider \(\mathcal {S}_{f^{\prime }}:F(X_1,X_2-a_{0,1}X_1/2 ,X_3,X_4-a_{0,1}X_3/2)=0\) and so the coefficient of \((X_1-X_3)(X_2-X_4)\) in \(F(X_1,X_2-a_{0,1}X_1/2 ,X_3,X_4-a_{0,1}X_3/2)\) vanishes, i.e. the coefficient \(a_{0,1}^{\prime }\) in \(f^{\prime }\) is 0. Therefore, by Theorem 7,

$$\begin{aligned} F(X_1,X_2-a_{0,1}X_1/2 ,X_3,X_4-a_{0,1}X_3/2)=(X_2 -X_4)^2+ (X_1-X_3)^{p^h+1}, \end{aligned}$$

otherwise \(\mathcal {S}_{f^{\prime }}\) (and so \(\mathcal {S}\)) contains an absolutely irreducible component defined over \({\mathbb {F}}_q\).

So, \(F(X_1,X_2,X_3,X_4)\) equals

$$\begin{aligned} (X_2 -X_4)^2 + (X_1-X_3)\left( (X_1-X_3)^{p^h}+a_{0,1}(X_2-X_4)+\frac{a_{0,1}^2}{4}(X_1-X_3)\right) . \end{aligned}$$
(5)

The claim follows. \(\square \)

2.2 Ovoids in Q(6, q)

Corollary 8 provides constraints on the shape of the functions \(f_1,f_2\) associated with putative ovoids \(O_6(f_1,f_2)\) of \(\textrm{Q}\).

Proposition 9

Suppose that

$$\begin{aligned}{} & {} (f_1(X,Y,Z), f_2(X,Y,Z))\ne \left( a_1(X)Y^{p^{h_1}}+\frac{c_1(X)^2Y}{4}+c_1(X)Z,a_2(Y)X^{p^{h_2}}\right. \nonumber \\{} & {} \quad \left. +\frac{c_2(Y)^2X}{4}+c_2(Y)Z\right) , \end{aligned}$$
(6)

for some \(a_1,c_1 \in {\mathbb {F}}_{q}[X]\), \(a_2,c_2\in {\mathbb {F}}_q[Y]\), \(h_1,h_2\ge 0\). If \(q>6.3 (d+1)^{13/3}\), \(\max \{\deg (f_1),\deg (f_2)\}=d\), then \(O_6(f_1,f_2)\) is not an ovoid.

Proof

Consider the variety \(\mathcal {W}^{\prime }_{f_1,f_2}:= \mathcal {W}_{f_1,f_2}\cap (X_5=X_2=\lambda )\), \(\lambda \in {\mathbb {F}}_q\), defined by

$$\begin{aligned} G^{\prime }_{f_1,f_2,\lambda }(X_0,X_1,X_3,X_4,X_6)=F_{f_1,f_2}(X_0,X_1,\lambda ,X_3,X_4,\lambda ,X_6)=0, \end{aligned}$$

where

$$\begin{aligned} G^{\prime }_{f_1,f_2,\lambda }(1,X_1,X_3,X_4,X_6):= & {} (X_3-X_6)^2 + (X_4-X_1)(f_2(X_4,\lambda ,X_6)-f_2(X_1,\lambda ,X_3)), \end{aligned}$$

where \(f_2\) satisfies (6). Suppose that

$$\begin{aligned} f_2(X,\lambda ,Z)\ne \alpha _{\lambda } X^{p^j}+\frac{\beta _{\lambda }^2}{4}X+\beta _{\lambda } Z, \end{aligned}$$
(7)

for any \(\alpha _{\lambda },\beta _{\lambda } \in {\mathbb {F}}_q\). Then, by Corollary 8, \(\mathcal {W}^{\prime }_{f_1,f_2}\) possesses an absolutely irreducible component defined over \({\mathbb {F}}_q\). Such a component, since \(q>6.3 (d+1)^{13/3}\), contains at least an \({\mathbb {F}}_q\)-rational point \((1,\overline{x}_1,\overline{x}_3,\overline{x}_4,\overline{x}_6)\), with \(\overline{x}_1\ne \overline{x}_4\) or \(\overline{x}_3\ne \overline{x}_6\). Therefore there exists a six-tuple \((1,\overline{x}_1,\lambda ,\overline{x}_3,\overline{x}_4,\lambda ,\overline{x}_6)\), with \((\overline{x}_1,\lambda ,\overline{x}_3)\ne (\overline{x}_4,\lambda ,\overline{x}_6)\) satisfying \(F_{f_1,f_2}(X_0,X_1,X_2,X_3,X_4,X_5,X_6)=0\) (defined as in (3)) and therefore \(O_6(f_1,f_2)\) is not an ovoid.

Suppose now that \(f_2(X,Y,Z)\) contains a term \(a_{i,j,k}X^iY^jZ^k\ne 0\), with \((i,k)\notin \{ (p^{h_2},0),(1,0),(0,1)\}\). The coefficient of \(X^iZ^k\) in \(f_2(X,\lambda ,Z)\) is determined by \(A_{i,k}(\lambda ):=\sum _{j}a_{i,j,k}\lambda ^j\) and, by (7), we have that \(A_{i,k}(\lambda )=0\) for each \(\lambda \in {\mathbb {F}}_q\). Since \(\deg A_{i,k}(Y)\le d<q\), \(A_{i,k}(Y)\equiv 0\) for each \((i,k)\notin \{ (p^{h_2},0),(1,0),(0,1)\}\). Also, \((A_{0,1}(\lambda ))^2=4A_{1,0}(\lambda )\) for each \(\lambda \in {\mathbb {F}}_q\). Since \(\deg (A_{0,1}(Y))^2\le 2d<q\), we have that \((A_{0,1}(Y))^2=4A_{1,0}(Y)\). Summing up,

$$\begin{aligned} f_2(X,Y,Z)=a_2(Y)X^{p^{h_2}}+\frac{c_2(Y)^2}{4}X+c_2(Y)Z, \end{aligned}$$

for some \(a_2,c_2\in {\mathbb {F}}_q[Y]\).

A similar argument, for any \(\lambda \in {\mathbb {F}}_q\), holds for \(\mathcal {W}^{\prime \prime }_{f_1,f_2}:= \mathcal {W}_{f_1,f_2}\cap (X_4=X_1=\lambda )\) defined by

$$\begin{aligned} G^{\prime \prime }_{f_1,f_2,\lambda }(X_0,X_2,X_3,X_5,X_6)=F_{f_1,f_2}(X_0,\lambda ,X_2,X_3,\lambda ,X_5,X_6)=0, \end{aligned}$$

that is

$$\begin{aligned}G^{\prime \prime }_{f_1,f_2,\lambda }(1,X_2,X_3,X_5,X_6):= & {} (X_3-X_6)^2 + (X_5-X_2)(f_1(\lambda ,X_5,X_6)-f_1(\lambda ,X_2,X_3)). \end{aligned}$$

So, if \(O_6(f_1,f_2)\) is an ovoid then

$$\begin{aligned} f_1(X,Y,Z)=a_1(X)Y^{p^{h_1}}+\frac{c_1(X)^2}{4}Y+c_1(X)Z, \end{aligned}$$

for some \(a_1,c_1\in {\mathbb {F}}_q[X]\). \(\square \)

By Proposition 9, if \(O_6(f_1,f_2)\) is an ovoid then the polynomials \(F(1,X_1,X_2,X_3,X_4,X_5,X_6)\) defining \(\mathcal {W}_{f_1,f_2}\) read

$$\begin{aligned}{} & {} (X_3-X_6)^2 + (X_4-X_1)\Bigg (a_2(X_5)X_4^{p^{h_2}}-a_2(X_2)X_1^{p^{h_2}}+\frac{c_2^2(X_5)}{4}X_4\nonumber \\{} & {} \quad -\frac{c_2^2(X_2)}{4}X_1 +c_2(X_5)X_6-c_2(X_2)X_3\Bigg )\nonumber \\{} & {} \quad +(X_5-X_2)\Bigg (a_1(X_4)X_5^{p^{h_1}}-a_1(X_1)X_2^{p^{h_1}}+\frac{c_1^2(X_4)}{4}X_5\nonumber \\{} & {} \quad -\frac{c_1^2(X_1)}{4}X_2 +c_1(X_4)X_6-c_1(X_1)X_3\Bigg ). \end{aligned}$$
(8)

Proposition 10

Suppose that \(\deg (\mathcal {W}_{f_1,f_2})=2\). Then \(O_6(f_1,f_2)\) is not an ovoid.

Proof

In this case (8) defines a quadric in \(\textrm{PG}(6,q)\) and, by Theorem 4, it contains at least \(q^4\) affine \({\mathbb {F}}_q\)-rational points. Thus, there exists at least an affine \({\mathbb {F}}_q\)-rational point off the solid \(X_1-X_4=X_2-X_5=X_3-X_6=0\) and, by Theorem 5, \(O_6(f_1,f_2)\) is not an ovoid. \(\square \)

In the sequel, we will make use of the following notations.

$$\begin{aligned} \deg (a_1)=A_1, \quad \deg (a_2)=A_2, \quad \deg (c_1)=C_1, \quad \deg (c_2)=C_2. \end{aligned}$$
(9)

When one of the above polynomials vanishes, we use \(-\infty \) to indicate its degree.

Proposition 11

Let \(p^{h_1}=p^{h_2}=1\). Then \(\mathcal {W}_{f_1,f_2}\) contains an absolutely irreducible component defined over \({\mathbb {F}}_q\).

Proof

By way of contradiction, suppose that \(O_6(f_1,f_2)\) is an ovoid. By Proposition 10, we can suppose that \(\deg (\mathcal {W}_{f_1,f_2})\ge 3\).

Now, \(F(1,X_1,X_2,X_3,X_4,X_5,X_6)\) reads

$$\begin{aligned}{} & {} (X_3-X_6)^2 + (X_4-X_1)\Bigg (\alpha _1(X_5)X_4-\alpha _1(X_2)X_1+ \beta _1(X_5)X_6-\beta _1(X_2)X_3\Bigg )\nonumber \\{} & {} \quad +(X_5-X_2)\Bigg (\alpha _2(X_4)X_5-\alpha _2(X_1)X_2 +\beta _2(X_4)X_6-\beta _2(X_1)X_3\Bigg ), \end{aligned}$$
(10)

for some polynomials \(\alpha _1,\alpha _2,\beta _1,\beta _2\) of degrees \(A_1,A_2,B_1,B_2\) (possibly \(-\infty \)). Note that at least one among \(A_1,A_2,B_1,B_2\) is positive, since \(\deg (\mathcal {W}_{f_1,f_2})\ge 3\).

We distinguish a number of cases, analyzing the homogeneous component \(M(X_1,X_2,X_3,X_4,X_5,X_6)\) of highest degree, say \(\ell \), in \(F(1,X_1,X_2,X_3,X_4,X_5,X_6)\).

  1. C.1.

    \(\ell =A_1+2\ge 3\).

    Then \(A_1>0\). The component of highest degree \(N(X_1,X_4,X_5,X_6)\) in \(M(X_1,1,1,X_4,X_5,X_6)\) is

    $$\begin{aligned} u (X_4-X_1)X_5^{A_1}X_4+v (X_4-X_1)X_5^{B_1}X_6+ wX_4^{A_2} X_5^2+z X_4^{B_2}X_5X_6, \end{aligned}$$

    for some \(u,v,w,z\in {\mathbb {F}}_q\), \(u\ne 0\). It is already seen that \(\deg _{X_1}(N(X_1,X_4,X_5,X_6))=1\) and therefore \(N(X_1,X_4,X_5,X_6)=0\) contains a nonrepeated absolutely irreducible component defined over \({\mathbb {F}}_q\). Such a component extends to an absolutely irreducible component defined over \({\mathbb {F}}_q\) in \(\mathcal {W}_{f_1,f_2}\) by applying twice Lemma 6.

  2. C.2.

    \(\ell =B_1+2\ge 3\) and \(B_1>A_1\).

    Then \(B_1>0\). The component of highest degree \(N(X_1,X_4,X_5,X_6)\) in \(M(X_1,1,1,X_4,X_5,X_6)\) is

    $$\begin{aligned} v (X_4-X_1)X_5^{B_1}X_6+ wX_4^{A_2} X_5^2+z X_4^{B_2}X_5X_6, \end{aligned}$$

    for some \(v,w,z\in {\mathbb {F}}_q\), \(v\ne 0\), and \(\deg _{X_1}(N(X_1,X_4,X_5,X_6))=1\). The claim follows similar as in Case C.1.

  3. C.3.

    \(\ell =B_2+2\ge 3\), and \(B_2>A_1\), \(B_2>B_1\) .

    Then \(B_2>0\). The component of highest degree \(N(X_2,X_4,X_5,X_6)\) in \(M(1,X_2,1,X_4,X_5,X_6)\) is

    $$\begin{aligned} (X_5-X_2)(w X_4^{A_2}X_5+ zX_4^{B_2} X_6), \end{aligned}$$

    for some \(w,z\in {\mathbb {F}}_q\), \(z\ne 0\), and \(\deg _{X_6}(N(X_2,X_4,X_5,X_6))=1\). The claim follows similar as in Case C.1.

  4. C.4.

    \(\ell =A_2+2\ge 3\) and \(A_2>A_1\), \(A_2>B_1\), \(A_2>B_2\).

    Then \(A_2>0\). The component of highest degree \(N(X_2,X_4,X_5,X_6)\) in \(M(1,X_2,1,X_4,X_5,X_6)\) is \(w(X_5-X_2)X_4^{A_2}X_5\), with \(w\in {\mathbb {F}}_q^*\). The claim follows similar as in Case C.1.

In all the cases above we obtain that \(\mathcal {W}_{f_1,f_2}\) contains an absolutely irreducible component defined over \({\mathbb {F}}_q\). \(\square \)

From now on we will make use a number of times of the following lemma to show that a polynomial \(F(X_1,\ldots ,X_n)\) is not square in \(\overline{{\mathbb {F}}_q}[X_1,\ldots ,X_n]\).

Lemma 12

Let \(F(X_1,\ldots ,X_n)\) be a polynomial in \({{\mathbb {F}}_q}[X_1,\ldots ,X_n]\). Assume that \(F(X_1,\ldots ,X_n)\) is a square in \(\overline{{\mathbb {F}}_q}[X_1,\ldots ,X_n]\). Then, the following hold:

  • any specialization \(X_i=\alpha _i\) of \(F(X_1,\ldots ,X_n)\) is a square.

  • the homogeneous part in \(F(X_1,\ldots ,X_n)\) of highest degree is a square too.

  • if the homogeneous parts of highest and second highest degree in \(F(X_1,\ldots ,X_n)\) are \(G(X_1,\ldots ,X_n)^{\ell }\) and \(H(X_1,\ldots ,X_n)\), for some even number \(\ell \), then \(G(X_1,\ldots ,X_n)^{\ell /2}\) divides \(H(X_1,\ldots ,X_n)\).

Proof

By hypothesis, there is a polynomial \(W(X_1,\ldots ,X_n)\) such that \(F(X_1,\ldots ,X_n)=(W(X_1,\ldots ,X_n))^2\). It follows immediately that any specialization \(X_i=\alpha _i\) of \(F(X_1,\ldots ,X_n)\) is a square. Now, write

$$\begin{aligned} W(X_1,\ldots ,X_n)=\sum _{i \le m}L_i(X_1,\ldots ,X_n), \end{aligned}$$

where m denotes the degree of \(W(X_1,\ldots ,X_n)\) and \(L_i(X_1,\ldots ,X_n)\) is the homogeneous part of \(W(X_1,\ldots ,X_n)\) of degree i. Then, we have

$$\begin{aligned} F(X_1,\ldots ,X_n)=\sum _{i \le m}(L_i(X_1,\ldots ,X_n))^2 + 2 \sum _{i \ne j}L_i(X_1,\ldots ,X_n)L_j(X_1,\ldots ,X_n), \end{aligned}$$

and so \(F(X_1,\ldots ,X_n)\) has degree 2m. The homogeneous parts of highest and second highest degree in \(F(X_1,\ldots ,X_n)\) are given by

$$\begin{aligned} (L_m(X_1,\ldots ,X_n))^2 \,\,\, \textrm{and} \,\,\, 2L_m(X_1,\ldots ,X_n)L_{k}(X_1,\ldots ,X_n), \end{aligned}$$

respectively, where \(k=\max \{ i : L_i(X_1,\ldots ,X_n) \not \equiv 0, i \ne m\}\). The claim follows. \(\square \)

Proposition 13

Let

$$\begin{aligned} \Delta (X_1,X_2,X_4,X_5):= & {} \Big ((X_4-X_1)c_2(X_5)+(X_5-X_2)c_1(X_4)\Big )^2\nonumber \\{} & {} -4(X_4-X_1)\left( a_2(X_5)X_4^{p^{h_2}}-a_2(X_2)X_1^{p^{h_2}}+\frac{c_2^2(X_5)}{4}X_4-\frac{c_2^2(X_2)}{4}X_1 \right) \nonumber \\{} & {} -4(X_5-X_2)\left( a_1(X_4)X_5^{p^{h_1}}-a_1(X_1)X_2^{p^{h_1}}+\frac{c_1^2(X_4)}{4}X_5-\frac{c_1^2(X_1)}{4}X_2 \right) \nonumber \\ \end{aligned}$$
(11)

If \(\Delta \) is not a square in \(\overline{{\mathbb {F}}_q}[X_1,X_2,X_4,X_5]\) then \(\mathcal {W}_{f_1,f_2}\) contains an absolutely irreducible component defined over \({\mathbb {F}}_q\).

Proof

Consider the hypersurface \(\mathcal {W}^{\prime }_{f_1,f_2}:= \mathcal {W}_{f_1,f_2}\cap (X_3=0)\). Note that, since the polynomial \(F(1,X_1,X_2,0,X_4,X_5,X_6)\) is of degree 2 in \(X_6\), \(\mathcal {W}^{\prime }_{f_1,f_2}\) is absolutely irreducible if and only if the discriminant \(\Delta (X_1,X_2,X_4,X_5)\) with respect to \(X_6\) is a non-square in \(\overline{{\mathbb {F}}_q}[X_1,X_2,X_4,X_5]\). Therefore, if \(\Delta (X_1,X_2,X_4,X_5)\) is not a square in \(\overline{{\mathbb {F}}_q}[X_1,X_2,X_4,X_5]\) then \(\mathcal {W}^{\prime }_{f_1,f_2}\) is absolutely irreducible and by Lemma 6 the hypersurface \(\mathcal {W}_{f_1,f_2}\) contains an absolutely irreducible component defined over \({\mathbb {F}}_q\). \(\square \)

Proposition 14

Let \(p^{h_1}=1\) and \(p^{h_2}\ge 3\) or \(p^{h_1}\ge 3\) and \(p^{h_2}=1\). Then \(\mathcal {W}_{f_1,f_2}\) contains an absolutely irreducible component defined over \({\mathbb {F}}_q\).

Proof

Suppose by way of contradiction that \(\mathcal {W}_{f_1,f_2}\) does not contain an absolutely irreducible component defined over \({\mathbb {F}}_q\).

Without loss of generality, we can suppose that \(p^{h_1}\ge 3\) and \(p^{h_2}=1\). Let \(\widetilde{a_2}(X)=a_2(X)+\frac{c_2^2(X)}{4}\), with \(\deg (\widetilde{a_2})=\widetilde{A_2}\). Now, the polynomial \(F(1,X_1,X_2,X_3,X_4,X_5,X_6)\) reads

$$\begin{aligned}{} & {} (X_3-X_6)^2+ (X_4-X_1)\Bigg (\widetilde{a_2}(X_5)X_4-\widetilde{a_2}(X_2)X_1 +c_2(X_5)X_6-c_2(X_2)X_3\Bigg )\nonumber \\{} & {} \quad +(X_5-X_2)\Bigg (a_1(X_4)X_5^{p^{h_1}}-a_1(X_1)X_2^{p^{h_1}}+\frac{c_1^2(X_4)}{4}X_5 -\frac{c_1^2(X_1)}{4}X_2 +c_1(X_4)X_6-c_1(X_1)X_3\Bigg ). \end{aligned}$$

First, we claim that the homogeneous part \(M(X_1,X_2,X_3,X_4,X_5,X_6)\) of highest degree in the polynomial \(F(1,X_1,X_2,X_3,X_4,X_5,X_6)\) is a linear combination (with not all the coefficients zero) of

$$\begin{aligned} t_1= & {} (X_4-X_1)(X_5^{\widetilde{A_2}}X_4-X_2^{\widetilde{A_2}}X_1),\\ t_2= & {} (X_4-X_1)(X_5^{C_2}X_6-X_2^{C_2}X_3),\\ t_3= & {} (X_5-X_2)(X_4^{A_1}X_5^{p^{h_1}}-X_1^{A_1}X_2^{p^{h_1}}),\\ t_4= & {} (X_5-X_2)(X_4^{2C_1}X_5-X_1^{2C_1}X_2). \end{aligned}$$

To see this, it is enough to observe that any such linear combination cannot vanish since each of the following monomials

$$\begin{aligned} X_1X_4X_5^{\widetilde{A_2}}, X_1X_6X_5^{C_2},X_1^{A_1}X_2^{p^{h_1}}X_5,X_1^{2C_1}X_2X_5 \end{aligned}$$

is contained in a unique \(t_i\). Also, if \(t_1\), \(t_2\) or \(t_4\) are nonzero then \(\widetilde{A_2}\), \(C_2\), or \(C_1\) (respectively) are nonnegative, since \(\deg (\mathcal {W}_{f_1,f_2})>3\).

Thus \(M(X_1,X_2,X_3,X_4,X_5,X_6)=u_1t_1+u_2t_2+u_3t_3+u_4t_4\) for some \(u_i\in {\mathbb {F}}_q\), not all vanishing.

If \(u_2\ne 0\), then \(M(X_1,X_2,X_3,X_4,X_5,X_6)\) is of degree one in \(X_6\) and therefore the hypersurface \(M(X_1,X_2,X_3,X_4,X_5,X_6)=0\) contains a nonrepeated absolutely irreducible component defined over \({\mathbb {F}}_q\). Such a component extends to an absolutely irreducible component defined over \({\mathbb {F}}_q\) in \(\mathcal {W}_{f_1,f_2}\) by applying Lemma 6, a contradiction.

From now on we can suppose that \(u_2=0\). If \(u_1\ne 0\), then the part of the highest degree in \(M(X_1,X_2,X_3,X_4,X_5,X_6)\) is of degree one in \(X_1\) and it contains a nonrepeated absolutely irreducible component defined over \({\mathbb {F}}_q\). Such a component extends to an absolutely irreducible component defined over \({\mathbb {F}}_q\) in \(\mathcal {W}_{f_1,f_2}\) by applying twice Lemma 6, a contradiction.

Thus, only \(u_3\) and \(u_4\) can be nonzero. If \(u_4\ne 0\) or \(u_4=0\) and \(A_1\ne 0\) then (since \(C_1>0\)) \(X_5-X_2\) is a nonrepeated absolutely irreducible factor defined over \({\mathbb {F}}_q\) of \(M(X_1,X_2,X_3,X_4,X_5,X_6)\). Arguing as above we get a contradiction.

So \(u_1=u_2=u_4=0\), \(u_3\ne 0\), \(A_1=0\), and \(M(X_1,X_2,X_3,X_4,X_5,X_6)=u_3(X_5-X_2)^{p^{h_1}+1}\). Note that \(p^{h_1}+1>\max \{2C_1+2,C_2+2\}.\)

We now investigate the discriminant \(\Delta (X_1,X_2,X_4,X_5)\) of \(F(1,X_1,X_2,0,X_4,X_5,X_6)\) with respect to \(X_6\). Recall that, since \(\mathcal {W}_{f_1,f_2}\) does not contain an absolutely irreducible component defined over \({\mathbb {F}}_q\), \(\Delta (X_1,X_2,X_4,X_5)\) must be a square.

By direct computations, \(\Delta (X_1,X_2,X_4,X_5)\) reads

$$\begin{aligned}{} & {} ((X_4-X_1)c_2(X_5)+(X_5-X_2)c_1(X_4))^2-4(X_4-X_1)(a_2(X_5)X_4-a_2(X_2)X_1)\\{} & {} \quad -(X_4-X_1)( c_2^2(X_5)X_4-c_2^2(X_2)X_1)-4\alpha (X_5-X_2)^{p^{h_1}+1} \\{} & {} \quad -(X_5-X_2)( c_1^2(X_4)X_5-c_1^2(X_1)X_2)\\= & {} -4\alpha (X_5-X_2)^{p^{h_1}+1}+(X_4-X_1)X_1(c_2^2(X_2)-c_2^2(X_5))+(X_5-X_2)X_2(c_1^2(X_1)-c_1^2(X_4))\\{} & {} \quad +2(X_4-X_1)(X_5-X_2)c_2(X_5)c_1(X_4)-4 (X_4-X_1)(a_2(X_5)X_4-a_2(X_2)X_1), \end{aligned}$$

for some \(\alpha \ne 0\).

Clearly, the homogeneous component of highest degree in \(\Delta \) is \(-4\alpha (X_5-X_2)^{p^{h_1}+1}\), which is a square. Now we claim that the homogeneous part \(N(X_1,X_2,X_4,X_5)\) of the second highest degree in \(\Delta (X_1,X_2,X_4,X_5)\) is a linear combination of

$$\begin{aligned} z_1:= & {} (X_4-X_1)X_1(X_2^{2C_2}-X_5^{2C_2}),\\ z_2:= & {} (X_5-X_2)X_2(X_1^{2C_1}-X_4^{2C_1}),\\ z_3:= & {} (X_4-X_1)(X_5^{A_2}X_4-X_2^{A_2}X_1),\\ z_4:= & {} (X_4-X_1)(X_5-X_2)X_5^{C_2}X_4^{C_1}.\\ \end{aligned}$$

We can suppose that \(C_1,C_2>0\), otherwise we just exclude \(z_1\) and \(z_2\).

If \(C_1=0\) and \(C_2>0\), then \(z_2\equiv 0\), \(\deg (z_1)>\deg (z_4)\), and any linear combination of \(z_1\) and \(z_3\) does not vanish. A similar argument applies to the case \(C_1>0\) and \(C_2=0\). If \(C_1=C_2=0\) then \(z_1\equiv z_2\equiv 0\) and any linear combination of \(z_3\) and \(z_4\) does not vanish.

So we consider only \(C_1,C_2>0\).

  1. 1.

    \((C_1,A_2)\ne (1,C_2+1)\). The four monomials \(X_1^2X_5^{2C_2}, X_2^2X_4^{2C_1},X_4^2X_5^{A_2},X_1X_2X_4^{C_1}X_5^{C_2}\) belong to a unique \(z_i\) and therefore a nontrivial linear combination of \(z_i\) cannot vanish.

  2. 2.

    \((C_1,A_2)=(1,C_2+1)\). If \(C_2>1\) the degree of \(z_1\) is the highest among the \(z_i\)’s.

  3. 3.

    \((C_1,C_2,A_2)=(1,1,2)\). If \(\lambda _1z_1+\lambda _2z_2+\lambda _3z_3+\lambda _4z_4=0\) then \(\lambda _3=0\) since \((X_5-X_2)\) divides \(z_1,z_2,z_4\) but not \(z_3\). Now, it can be readily seen that \(\lambda _1z_1+\lambda _2z_2+\lambda _4z_4\) cannot vanish for any \((\lambda _1,\lambda _2,\lambda _4)\ne (0,0,0)\).

Thus we know that

$$\begin{aligned} N(X_1,X_2,X_4,X_5)=\delta _1 w_1^2 z_1+\delta _2w_2^2 z_2+2w_1w_2 \delta _4z_4+\delta _3w_3z_3, \end{aligned}$$

for some \(w_1,w_2,w_3\in {\mathbb {F}}_q^*\), and \(\delta _1,\delta _2,\delta _3,\delta _4\in \{0,1\}\), where \(\delta _i=1\) only if the corresponding \(z_i\) has the highest degree.

Recall that since \(\Delta (X_1,X_2,X_4,X_5)\) is a square, \((X_5-X_2)^{(p^{h_1}+1)/2}\mid N(X_1,X_2,X_4,X_5)\) and \((p^{h_1}+1)/2\ge 2\); see Lemma 12.

If \(C_1=C_2=0\) then \(N(X_1,X_2,X_4,X_5)=\delta _3w_3 (X_4-X_1)(X_5^{A_2}X_4-X_2^{A_2}X_1)+2\delta _4 w_1w_2(X_4-X_1)(X_5-X_1)\) which is never divisible by \((X_5-X_2)^2\).

If \(C_1=0\) and \(C_2>0\) then \(N(X_1,X_2,X_4,X_5)\) is

$$\begin{aligned}{} & {} \delta _1w_1^2(X_4-X_1)X_1(X_2^{2C_2}-X_5^{2C_2})+\delta _3 w_3 \\{} & {} \quad (X_4-X_1)(X_5^{A_2}X_4-X_2^{A_2}X_1)+2\delta _4w_1w_2(X_4-X_1)(X_5-X_1)X_5^{C_2}, \end{aligned}$$

which is divisible by \((X_5-X_2)\) only if \(\delta _3=0\). Also \(C_2>0\) yields \(\delta _4=0\) since \(\deg (z_1)>\deg (z_4)\). This forces \(p\mid 2C_2\).

Analogously, \(C_1>0\) and \(C_2=0\) yields a contradiction.

Suppose now that \(C_1,C_2>0\). Since \((X_5-X_2)\) must divide \(N(X_1,X_2,X_4,X_5)\), \(\delta _3=0\). Now \((X_5-X_2)^2\) divides \(N(X_1,X_2,X_4,X_5)\) only if

$$\begin{aligned}{} & {} \phi (X_1,X_2,X_4):=2C_2\delta _1 w_1^2(X_4-X_1)X_1X_2^{2C_2-1}+\delta _2 w_2^2X_2(X_1^{2C_1}\\{} & {} \quad -X_4^{2C_1})+2\delta _4w_1w_2(X_4-X_1)X_2^{C_2}X_4^{C_1} \end{aligned}$$

is the zero-polynomial.

If \(\delta _2\ne 0\), then, considering \(X_4=0\), we get

$$\begin{aligned} \phi (X_1,X_2,X_4)=-2C_2\delta _1w_1^2X_1^2X_2^{2C_2-1}+\delta _2w_2^2X_2X_1^{2C_1}, \end{aligned}$$

which does not vanish unless \(C_1=C_2=1\). In this case \(\delta _1=\delta _2=\delta _4=1\) and

$$\begin{aligned} \phi (X_1,X_2,X_4)=2 w_1^2(X_4-X_1)X_1X_2+ w_2^2X_2(X_1^{2}-X_4^{2})+2w_1w_2(X_4-X_1)X_2X_4 \end{aligned}$$

vanishes only if \(2w_1^2=2w_1w_2\), \(2w_1^2=w_2^2\), which yields \(w_1=w_2=0\), a contradiction.

So we can suppose that \(\delta _2=0\) and

$$\begin{aligned} \phi (X_1,X_2,X_4)=2C_2\delta _1 w_1^2(X_4-X_1)X_1X_2^{2C_2-1}+2\delta _4w_1w_2(X_4-X_1)X_2^{C_2}X_4^{C_1}. \end{aligned}$$

It is readily seen that \(\phi \) vanishes only if \(\delta _4=0\) and \(p \mid 2C_2\).

Summing up, \(N(X_1,X_2,X_4,X_5)\) is divisible by \((X_2-X_5)^{(p^{h_1}+1)/2\ge 2}\) only if \(N(X_1,X_2,X_4,X_5)=(X_4-X_1)X_1(X_2^{2C_2}-X_5^{2C_2})\) with \(p\mid 2C_2\). Let \(\ell \) be the highest power of p dividing \(2C_2\). From \(p^{h_1}+1>\max \{2C_1+2,C_2+2\}\), \(\ell \le h_1-1\) and \((X_5-X_2)^{p^\ell +1}\) does not divide \(N(X_1,X_2,X_4,X_5)\). A final contradiction arises from \((p^{h_1}+1)/2\le p^{\ell }\le p^{h_1-1}\). The claim follows from Proposition 13. \(\square \)

By Proposition 14, if \(\mathcal {W}_{f_1,f_2}\) does not contain an absolutely irreducible component defined over \({\mathbb {F}}_q\) then both \(p^{h_1}>1\) and \(p^{h_2}>1\) and \(\deg (\mathcal {W}_{f_1,f_2})\ge 4\).

Proposition 15

Let \(\mathcal {W}_{f_1,f_2}\) be the hypersurface defined as in (8). Suppose that \(\deg (\mathcal {W}_{f_1,f_2})\ge 4\) and \(p^{h_1},p^{h_2}>1\). If \(A_1A_2> 0\) then \(\mathcal {W}_{f_1,f_2}\) contains an absolutely irreducible component defined over \({\mathbb {F}}_q\).

Proof

We proceed as in the proof of Proposition 14. Consider the variety \(\mathcal {W}^{\infty }_{f_1,f_2}:= \mathcal {W}_{f_1,f_2}\cap (X_0=0)\). It is defined by the homogeneous part \(L(X_1,X_2,X_3,X_4,X_5,X_6)\) of highest degree in (8) and it arises from a linear combination (with not all the coefficients zero) of

$$\begin{aligned} t_1= & {} (X_4-X_1)(X_5^{A_2}X_4^{p^{h_2}}-X_2^{A_2}X_1^{p^{h_2}}),\\ t_2= & {} (X_4-X_1)(X_5^{2C_2}X_4-X_2^{2C_2}X_1),\\ t_3= & {} (X_5-X_2)(X_4^{A_1}X_5^{p^{h_1}}-X_1^{A_1}X_2^{p^{h_1}}),\\ t_4= & {} (X_5-X_2)(X_4^{2C_1}X_5-X_1^{2C_1}X_2). \end{aligned}$$

To see this, it is enough to observe any such linear combination cannot vanish since each of the following monomials

$$\begin{aligned} X_1^{p^{h_2}}X_2^{A_2}X_4, X_1X_2^{2C_2}X_4,X_1^{A_1}X_2^{p^{h_1}}X_5,X_1^{2C_1}X_2X_5 \end{aligned}$$

is contained in a unique \(t_i\).

Thus

$$\begin{aligned} L(X_1,X_2,X_3,X_4,X_5,X_6)=\delta _1u_1t_1+\delta _2u_2t_2+\delta _3 u_3 t_3+\delta _4 u_4 t_4, \end{aligned}$$

with \(\delta _i \in \{0,1\}\), \(u_i \in {\mathbb {F}}_q^*\), and not all the \(\delta _i\)’s vanish. We distinguish two cases.

  1. (i)

    \((\delta _1,\delta _2)\ne (0,0)\). The homogeneous part in \(L(X_1,1,X_3,X_4,X_5,X_6)\) of highest degree is

    $$\begin{aligned} M{} & {} := \delta _1 u_1(X_4-X_1)X_5^{A_2}X_4^{p^{h_2}}+\delta _2u_2(X_4-X_1)X_5^{2C_2}X_4 +\delta _3 u_3 X_4^{A_1}X_5^{p^{h_1+1}}\\{} & {} \quad \ +\delta _4 u_4 X_5^2 X_4^{2C_1}, \end{aligned}$$

    which is of degree 1 in \(X_1\). Thus there exists a non-repeated \({\mathbb {F}}_q\)-rational factor of M (depending on \(X_1\)) which is absolutely irreducible. Therefore \(L(X_1,1,X_3,X_4,X_5,X_6)\) contains a non-repeated absolutely irreducible factor defined over \({\mathbb {F}}_q\), by Lemma 6. Again by Lemma 6 the corresponding variety extends to a non-repeated \({\mathbb {F}}_q\)-rational absolutely irreducible component in \(\mathcal {W}^{\infty }_{f_1,f_2}\).

  2. (ii)

    \((\delta _1,\delta _2)= (0,0)\). So, \((\delta _3,\delta _4)\ne (0,0)\). The homogeneous part in \(L(1,X_2,X_3,X_4,X_5,X_6)\) of highest degree is

    $$\begin{aligned} M:= \delta _3 u_3 (X_5-X_2) X_4^{A_1}X_5^{p^{h_1}} +\delta _4 u_4 (X_5-X_2) X_4^{2C_1}X_5, \end{aligned}$$

    which is of degree 1 in \(X_2\). Thus we obtain the same conclusion as above.

\(\square \)

Proposition 16

Let \(\mathcal {W}_{f_1,f_2}\) be the hypersurface defined as in (8). Suppose that \(\deg (\mathcal {W}_{f_1,f_2})\ge 4\) and \(p^{h_1},p^{h_2}>1\). If \((A_1,A_2)\notin \{(0,-\infty ),(-\infty ,0), (-\infty ,-\infty ),(0,0)\}\) then the hypersurface \(\mathcal {W}_{f_1,f_2}\) contains an absolutely irreducible component defined over \({\mathbb {F}}_q\).

Proof

Suppose that \(A_1> 0\), the case \(A_2> 0\) follows completely similar. In view of Proposition 15, we only have to consider \(A_2\le 0\).

Following the same notation as in Proposition 15, let \(L(X_1,X_2,X_3,X_4,X_5,X_6)\) be the part of the highest degree in (8). It arises from a linear combination (with not all the coefficients zero) \(\delta _1 u_1 t_1+\delta _2u_2 t_2+\delta _3u_3t_3+\delta _4 u_4 t_4\), where

$$\begin{aligned} t_1= & {} (X_4-X_1)^{p^{h_2}+1},\\ t_2= & {} (X_4-X_1)(X_5^{2C_2}X_4-X_2^{2C_2}X_1),\\ t_3= & {} (X_5-X_2)(X_4^{A_1}X_5^{p^{h_1}}-X_1^{A_1}X_2^{p^{h_1}}),\\ t_4= & {} (X_5-X_2)(X_4^{2C_1}X_5-X_1^{2C_1}X_2). \end{aligned}$$

Note that if \(A_2=-\infty \), that is \(a_2\equiv 0\), we may assume \(\delta _1=0\).

To see this, it is enough to observe any such linear combination cannot vanish since each of the following monomials

$$\begin{aligned} X_1^{p^{h_2}+1}, X_1X_2^{2C_2}X_4,X_1^{A_1}X_2^{p^{h_1}}X_5,X_1^{2C_1}X_2X_5 \end{aligned}$$

is contained in a unique \(t_i\).

Suppose that \((\delta _3,\delta _4)\ne (0,0)\). The homogeneous part in \(L(1,X_2,X_3,X_4,X_5,X_6)\) of highest degree is

$$\begin{aligned} M:= \delta _1 u_1X_4^{p^{h_2}+1}+\delta _2u_2X_4^2X_5^{2C_2} +\delta _3 u_3 (X_5-X_2) X_4^{A_1}X_5^{p^{h_1}} +\delta _4 u_4 (X_5-X_2) X_4^{2C_1}X_5, \end{aligned}$$

which is of degree 1 in \(X_2\). Thus there exists a non-repeated \({\mathbb {F}}_q\)-rational factor of M (depending on \(X_1\)) which is absolutely irreducible. Therefore \(L(1,X_2,X_3,X_4,X_5,X_6)\) contains a non-repeated absolutely irreducible factor defined over \({\mathbb {F}}_q\), by Lemma 6. Again by Lemma 6 the corresponding variety extends to a non-repeated \({\mathbb {F}}_q\)-rational absolutely irreducible component in \(\mathcal {W}^{\infty }_{f_1,f_2}\). Thus \((\delta _3,\delta _4)= (0,0)\).

If \(\delta _1=0\) or \(\delta _1\delta _2\ne 0\) then \(C_2>0\) and \((X_4-X_1)\) is a non-repeated \({\mathbb {F}}_q\)-rational absolutely irreducible factor of \(L(X_1,X_2,X_3,X_4,X_5,X_6)\) and, arguing as above, \(\mathcal {W}_{f_1,f_2}\) contains an absolutely irreducible component defined over \({\mathbb {F}}_q\).

Thus, we have only to deal with the case \(\delta _1\ne 0\) (which implies \(A_2= 0\)) and \(\delta _2=\delta _3=\delta _4=0\). Note that \(p^{h_2}+1>\max \{ 2C_2+2,A_1+p^{h_1}+1,2C_1+2\}\).

Consider now \(\Delta (X_1,X_2,X_4,X_5)\) defined as in (11) and we will prove that it is not a square in \(\overline{{\mathbb {F}}_q}[X_1,X_2,X_4,X_5]\) using Lemma 12.

Suppose by way of contradiction that \(\Delta (X_1,X_2,X_4,X_5)\) is a square in \(\overline{{\mathbb {F}}_q}[X_1,X_2,X_4,X_5]\).

Hence, \(\Delta (X_1,X_2,X_4,0)\) is a square in \(\overline{{\mathbb {F}}_q}[X_1,X_2,X_4]\). Recalling that \(p^{h_1}\ge 3\), it is readily seen that its homogeneous component of the second largest degree is

$$\begin{aligned} N(X_1,X_2,X_4)= & {} \delta _1 u_1 X_2^2 X_4^{2C_1}+\delta _2 u_2(X_4-X_1)X_1X_2^{2C_2}+\delta _3 u_3 X_2^{p^{h_1}+1}X_1^{A_1}-\delta _1 u_1 X_2^2X_1^{2C_1}, \end{aligned}$$

for some \(\delta _1,\delta _2,\delta _3 \in \{0,1\}\) not all vanishing, and \(u_1,u_2,u_3\in {\mathbb {F}}_q\). By Lemma 12, \((X_1-X_4)^{(p^{h_2}+1)/2}\) must divide \(N(X_1,X_2,X_4)\). This immediately forces \(\delta _3=0\) and thus \(p^{h_1}+1+A_1<\min \{2+2C_1,2+2C_2\}\).

Also, \(N^{\prime }(X_1,X_2,X_4):=N(X_1,X_2,X_4)/(X_4-X_1)\) reads

$$\begin{aligned} \delta _1 u_1 X_2^2 \left( \sum _{i=0}^{2C_1-1}X_4^{i}X_1^{2C_1-1-i}\right) +\delta _2 u_2X_1X_2^{2C_2}. \end{aligned}$$

Now, \((X_4-X_1)^2\mid N(X_1,X_2,X_4)\) if and only if \(N^{\prime }(X_1,X_2,X_1)\equiv 0\), that is

$$\begin{aligned} \delta _1 u_1 (2C_1)X_2^2X_1^{2C_1-1} +\delta _2 u_2X_1X_2^{2C_2}\equiv 0. \end{aligned}$$

The case \(C_2=1=C_1\) must be excluded, since \(4\le p^{h_1+1}+A_1<\min \{2+2C_1,2+2C_2\}=4\) yields a contradiction. The only possibility is then \(\delta _2=0\), \(\delta _1=1\), \(p\mid C_1\). In this case \(N(X_1,X_2,X_4)=u_1X_2^2(X_4^{2C_1}-X_1^{2C_1})=u_1X_2^2(X_4^{C_1}-X_1^{C_1})(X_4^{C_1}+X_1^{C_1})\), which is not divisible by \((X_1-X_4)^{(p^{h_2}+1)/2}\), since \(2C_1<p^{h_2}-1\). The claim follows from Proposition 13. \(\square \)

Proposition 17

Let \(\mathcal {W}_{f_1,f_2}\) be the hypersurface defined as in (8). Suppose that \(\deg (\mathcal {W}_{f_1,f_2})\ge 4\) and \(p^{h_1},p^{h_2}>1\). If \((A_1,A_2)= (0,-\infty )\) or \((A_1,A_2)= (-\infty ,0)\) then the hypersurface \(\mathcal {W}_{f_1,f_2}\) contains an absolutely irreducible component defined over \({\mathbb {F}}_q\) or \(O(f_1,f_2)\) is not an ovoid.

Proof

We consider only the case \((A_1,A_2)= (0,-\infty )\), the case \((A_1,A_2)= (-\infty ,0)\) being similar.

Note that \(C_2\ne -\infty \), otherwise \(f_2\equiv 0 \).

If \(C_1=-\infty \) and \(C_2=0\) then \(\mathcal {W}_{f_1,f_2}\) reads

$$\begin{aligned} (X_3-X_6)^2 + (X_4-X_1)\Bigg (\frac{c_2^2}{4}(X_4-X_1) +c_2(X_6-X_3)\Bigg ) +a_1(X_5-X_2)^{p^{h_1}+1}=0 \end{aligned}$$

and contains all the points on \(X_5=X_2\), \(2(X_6-X_3)+c_2(X_4-X_1)=0\) and \(O(f_1,f_2)\) is not an ovoid.

Following the same notation as in Proposition 15, let \(L(X_1,X_2,X_3,X_4,X_5,X_6)\) be the part of the highest degree in (8). It arises from a linear combination (with not all the coefficients zero) \(\delta _2u_2 t_2+\delta _3u_3t_3+\delta _4 u_4 t_4\), where

$$\begin{aligned} t_2= & {} (X_4-X_1)(X_5^{2C_2}X_4-X_2^{2C_2}X_1),\\ t_3= & {} (X_5-X_2)^{p^{h_1}+1},\\ t_4= & {} (X_5-X_2)(X_4^{2C_1}X_5-X_1^{2C_1}X_2). \end{aligned}$$

To see this, it is enough to observe any such linear combination cannot vanish since each of the following monomials

$$\begin{aligned} X_1X_2^{2C_2}X_4,X_2^{p^{h_1}+1},X_1^{2C_1}X_2X_5 \end{aligned}$$

is contained in a unique \(t_i\).

If \(\delta _2\ne 0\) then \(C_2>0\) since \(\deg (\mathcal {W}_{f_1,f_2})>3\) and the homogeneous part in \(L(X_1,1,X_3,X_4,X_5,X_6)\) of highest degree is of degree 1 in \(X_1\). Arguing as in the previous propositions \(\mathcal {W}_{f_1,f_2}\) contains an absolutely irreducible component defined over \({\mathbb {F}}_q\). So \(\delta _2=0\). If \(\delta _4\ne 0\) then \(C_1>0\) and \((X_5-X_2)\) is a nonrepeated factor of \(L(X_1,X_2,X_3,X_4,X_5,X_6)\). Arguing as above, \(\mathcal {W}_{f_1,f_2}\) contains an absolutely irreducible component defined over \({\mathbb {F}}_q\).

Thus \(\delta _3\ne 0\) and \(\delta _2=\delta _4=0\), which yields \(p^{h_1}+1>\max \{2C_2+2,2C_1+2\}\).

We consider now \(\Delta (X_1,X_2,X_4,X_5)\) as in (11) and we show that it is not a square in \(\overline{{\mathbb {F}}_q}[X_1,X_2,X_4,X_5]\) by means of Lemma 12. We have that

$$\begin{aligned} \Delta (X_1,X_2,X_4,X_5)\hspace{-0.3 cm}&=\hspace{-0.3 cm}&\Big ((X_4-X_1)c_2(X_5)+(X_5-X_2)c_1(X_4)\Big )^2\\{} & {} -4(X_4-X_1)\left( \frac{c_2^2(X_5)}{4}X_4-\frac{c_2^2(X_2)}{4}X_1 \right) \\{} & {} -4(X_5-X_2)\left( a_1X_5^{p^{h_1}}-a_1X_2^{p^{h_1}}+\frac{c_1^2(X_4)}{4}X_5-\frac{c_1^2(X_1)}{4}X_2 \right) \\= & {} X_1(X_4-X_1)(c_2^2(X_2)-c_2^2(X_5))+X_2(X_5-X_2)(c_1^2(X_1)-c_1^2(X_4))+\\{} & {} \alpha (X_5-X_2)^{p^{h_1}+1}+2(X_5-X_2)(X_4-X_1)c_1(X_4)c_2(X_5). \end{aligned}$$

If \(C_1=-\infty \) and \(C_2>0\), \(\Delta (X_1,X_2,X_4,X_5)\) is not a square and, by Proposition 13, \(\mathcal {W}_{f_1,f_2}\) contains an absolutely irreducible component defined over \({\mathbb {F}}_q\).

So we can suppose that \(C_1,C_2\ge 0\).

If \(\Delta (X_1,X_2,X_4,X_5)\) is a square in \(\overline{{\mathbb {F}}_q}[X_1,X_2,X_4,X_5]\), then the homogeneous part of the highest degree in \(\Delta (X_1,X_2,X_4,X_5)\) is clearly \((X_5-X_2)^{p^{h_1}+1}\) and \(\Delta (0,X_2,X_4,X_5)\) is a square in \(\overline{{\mathbb {F}}_q}[X_2,X_4,X_5]\). Recalling that \(p^{h_1}\ge 3\), it is readily seen that its homogeneous component of the second largest degree is

$$\begin{aligned} N(X_2,X_4,X_5)= & {} \delta _1 u_1 X_2(X_5-X_2)X_4^{2C_1}+\delta _2 u_2(X_5-X_2)X_4^{C_1+1}X_5^{C_2}, \end{aligned}$$

for some \(\delta _1,\delta _2 \in \{0,1\}\) not all vanishing, and \(u_1,u_2\in {\mathbb {F}}_q\). By Lemma 12, \((X_5-X_2)^{(p^{h_1}+1)/2}\) must divide \(N(X_2,X_4,X_5)\). This immediately forces \(p^{h_1}=3\), \(C_1=C_2=1\), a contradiction to \(4=p^{h_1}+1>\max \{2C_2+2,2C_1+2\}\). Thus \(\Delta (X_1,X_2,X_4,X_5)\) is not a square in \(\overline{{\mathbb {F}}_q}[X_1,X_2,X_4,X_5]\) and, by Proposition 13, \(\mathcal {W}_{f_1,f_2}\) contains an absolutely irreducible component defined over \({\mathbb {F}}_q\). \(\square \)

Proposition 18

Let \(\mathcal {W}_{f_1,f_2}\) be the hypersurface defined as in (8). Suppose that \(\deg (\mathcal {W}_{f_1,f_2})\ge 4\), \((A_1,A_2)= (-\infty ,-\infty )\), and \(p^{h_1},p^{h_2}>1\). Then the hypersurface \(\mathcal {W}_{f_1,f_2}\) contains an absolutely irreducible component defined over \({\mathbb {F}}_q\).

Proof

Since \((A_1,A_2)=(- \infty , -\infty )\), the polynomial \(F(1,X_1,X_2,X_3,X_4,X_5,X_6)\) defining \(\mathcal {W}_{f_1,f_2}\) reads

$$\begin{aligned}{} & {} (X_3-X_6)^2 + (X_4-X_1)\Bigg (\frac{c_2^2(X_5)}{4}X_4-\frac{c_2^2(X_2)}{4}X_1 +c_2(X_5)X_6-c_2(X_2)X_3\Bigg )\nonumber \\{} & {} \quad +(X_5-X_2)\Bigg (\frac{c_1^2(X_4)}{4}X_5-\frac{c_1^2(X_1)}{4}X_2 +c_1(X_4)X_6-c_1(X_1)X_3\Bigg ). \end{aligned}$$
(12)

We can consider \(C_1,C_2\ge 0\) otherwise \(f_1\) or \(f_2\) vanishes. Also, either \(C_1\) or \(C_2\) is positive otherwise \(\mathcal {W}_{f_1,f_2}\) has degree 2.

Thus

$$\begin{aligned} \Delta (X_1,X_2,X_4,X_5)= & {} \Big ((X_4-X_1)c_2(X_5)+(X_5-X_2)c_1(X_4)\Big )^2\\{} & {} -4(X_4-X_1)\left( \frac{c_2^2(X_5)}{4}X_4-\frac{c_2^2(X_2)}{4}X_1 \right) \\{} & {} -4(X_5-X_2)\left( \frac{c_1^2(X_4)}{4}X_5-\frac{c_1^2(X_1)}{4}X_2 \right) \\= & {} X_1(X_4-X_1)(c_2^2(X_2)-c_2^2(X_5))+X_2(X_5-X_2)(c_1^2(X_1)-c_1^2(X_4))\\{} & {} +2(X_5-X_2)(X_4-X_1)c_2(X_5)c_1(X_4). \end{aligned}$$

If \(C_2>C_1\) (resp. \(C_1>C_2\)) then the highest-degree homogeneous part in \(\Delta (X_1,X_2,X_4,X_5)\) is \(\alpha ^2 X_1(X_4-X_1)(X_2^{2C_2}-X_5^{2C_2})\) (resp. \(\alpha ^2 X_2(X_5-X_2)(X_1^{2C_1}-X_4^{2C_1})\)) which is not a square. If \(C_2=C_1>1\) then the highest-degree homogeneous part in \(\Delta (X_1,X_2,0,0)\)

$$\begin{aligned} -\alpha ^2X_1^2X_2^{2C_2}-\beta ^2X_2^2X_1^{2C_1}=X_1^2X_2^2(-\alpha ^2X_2^{2C_1-2}-\beta ^2X_1^{2C_1-2}) \end{aligned}$$

is not a square.

If \(C_2=C_1=1\) then the highest-degree homogeneous part in \(\Delta (X_1,X_2,X_4,X_5)\)

$$\begin{aligned}{} & {} \alpha ^2X_1(X_4-X_1)(X_2^2-X_5^2)+\beta ^2X_2(X_5-X_2)(X_1^2-X_4^2)+2\alpha \beta (X_5-X_2)(X_4-X_1)X_4X_5 \\{} & {} \quad =(X_4-X_1)(X_5-X_2)(-\alpha ^2X_1(X_2+X_5)-\beta ^2X_2(X_1+X_4)+2\alpha \beta X_4X_5) \end{aligned}$$

is not a square, since \((X_5-X_2)\) does not divide \((-\alpha ^2X_1(X_2+X_5)-\beta ^2X_2(X_1+X_4)+2\alpha \beta X_4X_5)\) unless \(\alpha =\beta =0\), which is not possible. In all these cases, by Lemma 12, \(\Delta (X_1,X_2,X_4,X_5)\) is not a square and, by Proposition 13, \(\mathcal {W}_{f_1,f_2}\) contains an absolutely irreducible component defined over \({\mathbb {F}}_q\). \(\square \)

Proposition 19

Let \(\mathcal {W}_{f_1,f_2}\) be the hypersurface defined as in (8). Suppose that \(\deg (\mathcal {W}_{f_1,f_2})\ge 4\), \((A_1,A_2)= (0,0)\), and \(p^{h_1},p^{h_2}>1\). Then the hypersurface \(\mathcal {W}_{f_1,f_2}\) contains an absolutely irreducible component defined over \({\mathbb {F}}_q\) unless

$$\begin{aligned} p^{h_1}=p^{h_2}=3, \text { and } C_1,C_2\le 1. \end{aligned}$$

Proof

Similar as in Proposition 18, we have that

$$\begin{aligned} \Delta (X_1,X_2,X_4,X_5)= & {} \Big ((X_4-X_1)c_2(X_5)+(X_5-X_2)c_1(X_4)\Big )^2\nonumber \\{} & {} -4(X_4-X_1)\left( a_2 X_4^{p^{h_2}}-a_2 X_1^{p^{h_2}}+\frac{c_2^2(X_5)}{4}X_4-\frac{c_2^2(X_2)}{4}X_1 \right) \nonumber \\{} & {} -4(X_5-X_2)\left( a_1X_5^{p^{h_1}}-a_1X_2^{p^{h_1}}+\frac{c_1^2(X_4)}{4}X_5-\frac{c_1^2(X_1)}{4}X_2 \right) \\= & {} X_1(X_4-X_1)(c_2^2(X_2)-c_2^2(X_5))+X_2(X_5-X_2)(c_1^2(X_1)-c_1^2(X_4))\\{} & {} +2(X_5-X_2)(X_4-X_1)c_2(X_5)c_1(X_4)-4a_2(X_4-X_1)^{p^{h_2}+1}\\{} & {} -4a_1(X_5-X_2)^{p^{h_1}+1}. \end{aligned}$$

Thus,

$$\begin{aligned} \Delta (0,X_2,1,X_5)= & {} X_2(X_5-X_2)(c_1^2(0)-c_1^2(1)) +2(X_5-X_2)c_2(X_5)c_1(1)\\{} & {} -4a_2-4a_1(X_5-X_2)^{p^{h_1}+1}, \end{aligned}$$

where \(a_1a_2\ne 0\).

We distinguish a few cases:

  1. (i)

    \(c_1(1)=0\).

    Then \(\Delta (0,X_2,1,X_5)\) reads \(-4a_2-4a_1(X_5-X_2)^{p^{h_1}+1}+c_1^2(0)(X_5-X_2)X_2\) and it cannot be a square.

  2. (ii)

    \(c_1(1)\ne 0\).

  • If \(C_2= p^{h_1}>1\) then \(\Delta (0,X_2,1,X_5)\) cannot be a square since its highest-degree homogeneous part is \(-4(X_5-X_2)^{p^{h_1}+1}+2c_1(1)c_2(X_5)(X_5-X_2)\), a nonsquare.

  • If \(C_2> p^{h_1}\), then \(2c_1(1)c_2(X_5)(X_5-X_2)\), the highest-degree homogeneous part in \(\Delta (0,X_2,1,X_5)\) is not a square.

  • If \(C_2< p^{h_1}\), the highest-degree homogeneous part in \(\Delta (0,X_2,1,X_5)\) is \(-4a_1(X_5-X_2)^{p^{h_1}+1}\). Since no other homogeneous part in \(\Delta (0,X_2,1,X_5)\) is divisible by \((X_5-X_2)^{(p^{h_1}+1)/2}\) if \(p^{h_1}+1>4\), \(\Delta (0,X_2,1,X_5)\) is not a square if \(p^{h_1}>3\). Also, when \(p^{h_1}=3\), and \(C_2<p^{h_1}\), we must have \(C_2\le 1\), otherwise the second highest-degree homogeneous part in \(\Delta (0,X_2,1,X_5)\) is \(X_5^2(X_5-X_2)\) which is not divisible by \((X_5-X_2)^{(p^{h_1}+1)/2}= (X_5-X_2)^{2}\). By Lemma 12\(\Delta (0,X_2,1,X_5)\) is not a square.

This shows that \(\Delta (0,X_2,1,X_5)\) is not a square unless \(p^{h_1}=3\) and \(C_2\le 1\).

A similar argument applied to \(\Delta (X_1,0,X_4,1)\) shows that this is not a square unless \(p^{h_2}=3\) and \(C_1\le 1\).

So, unless \(p^{h_2}=3=p^{h_1}\) and \(C_1,C_2\le 1\), by Proposition 13, \(\mathcal {W}_{f_1,f_2}\) contains an absolutely irreducible component defined over \({\mathbb {F}}_q\). \(\square \)

In view of the previous propositions, if we assume that \(O(f_1,f_2)\) is an ovoid and thus \(\mathcal {W}_{f_1,f_2}\) does not contain any \({\mathbb {F}}_q\)-rational absolutely irreducible components, then

  1. 1.

    \(p^{h_1}= p^{h_2}=3\);

  2. 2.

    \(C_1,C_2\le 1\) (possibly \(-\infty \)),

  3. 3.

    \(a_1(X)=a_1\), \(a_2(X)=a_2\), \(a_1a_2\ne 0\).

Secondly, from now on we assume that

$$\begin{aligned} c_1(X)=CX+D, \quad c_2(X)=EX+F, \end{aligned}$$

for \(C,D,E,F \in {\mathbb {F}}_q\).

Proposition 20

If \(p^{h_1}= p^{h_2}=3\), \(C_1,C_2\le 1\) (possibly \(-\infty \)), and \(A_1=A_2=0\), then \(\mathcal {W}_{f_1,f_2}\) contains an absolutely irreducible component defined over \({\mathbb {F}}_q\), unless \((D,F)=(0,0)\), \(C=-E\), and \(a_2=E^4/a_1\). In this case, \(O_6(f_1,f_2)\) is an ovoid if and only if \(-a_1\) is not a square in \({\mathbb {F}}_q\).

Proof

We want to determine whether \(\Delta (X_1,X_2,X_4,X_5)\) is a square by applying Lemma 12. We have

$$\begin{aligned} \Delta (0,X_2,X_4,0)= & {} -(4a_1X_2^4 -C^2 X_2^2X_4^2 + 4a_2X_4^4) +2CX_2X_4(DX_2 -FX_4) -2FDX_2X_4\\= & {} -(a_1X_2^4 + 2C^2 X_2^2X_4^2 + a_2X_4^4) -CX_2X_4(DX_2 -FX_4) +FDX_2X_4. \end{aligned}$$

First note that \(C\ne 0\), otherwise \((4a_1X_2^4 +2C^2 X_2^2X_4^2 + 4a_2X_4^4)\) cannot be a square. Since \(FDX_2X_4\) must be a square, \(FD=0\). Now, in order to have that \(\Delta (0,X_2,X_4,0)\) is a square, \(-CX_2X_4(DX_2 -FX_4)\) must be a square and this can happen only if it vanishes, i.e. \(F=D=0\). Also, if \((a_1X_2^4 + 2C^2 X_2^2X_4^2 + a_2X_4^4)\) is a square, then \(C^4= a_1a_2\).

Now, \(\Delta (X_1,X_2,X_4,X_5)\) reads

$$\begin{aligned}{} & {} ( E(X_4-X_1) X_5+C(X_5-X_2)X_4)^2\\{} & {} \quad - (X_4-X_1) \left( a_2 (X_4- X_1)^3+E^2 X_5^2 X_4-E^2 X_2^2 X_1\right) \\{} & {} \quad -(X_5-X_2) \left( a_1 (X_5- X_2)^3+C^2 X_4^2 X_5-C^2 X_1^2 X_2\right) . \end{aligned}$$

In particular

$$\begin{aligned} a_1\Delta (X_1,0,X_4,1)= & {} -a_2(X_1 -X_4)^4 +Ea_1(E X_1 + C X_4)(X_1 -X_4) -a_1, \end{aligned}$$

which is a square only if \(C=-E\). In this case

$$\begin{aligned}{} & {} -a_1F(1,X_1,X_2,X_3,X_4,X_5,X_6)\\{} & {} =F_1(1,X_1,X_2,X_3,X_4,X_5,X_6)F_2(1,X_1,X_2,X_3,X_4,X_5,X_6), \end{aligned}$$

where

$$\begin{aligned} F_1(1,X_1,X_2,X_3,X_4,X_5,X_6):= & {} E^2 X_1^2 + E^2 X_1 X_4 + \eta ^2 \sqrt{a_1} E X_1 X_5 + a_1 X_2^2 - \eta ^2 \sqrt{a_1} E X_2 X_4\\{} & {} + a_1 X_2 X_5 - \eta ^2 \sqrt{a_1} X_3 + E^2X_4^2 + a_1 X_5^2 + \eta ^2 \sqrt{a_1} X_6;\\ F_2(1,X_1,X_2,X_3,X_4,X_5,X_6):= & {} E^2 X_1^2 + E^2 X_1 X_4 - \eta ^2 \sqrt{a_1} E X_1 X_5 + a_1 X_2^2 + \eta ^2 \sqrt{a_1} E X_2 X_4\\{} & {} + a_1 X_2 X_5 + \eta ^2 \sqrt{a_1} X_3 + E^2X_4^2 + a_1 X_5^2 - \eta ^2 \sqrt{a_1} X_6, \end{aligned}$$

with \(\eta \in {\mathbb {F}}_9\) satisfying \(\eta ^2 -\eta -1=0\).

If \(q=3^{2h}\) and \(a_1\in \square _q\) or \(q=3^{2h+1}\) and \(a_1\notin \square _q\) then both the factors \(F_1\) and \(F_2\) are defined over \({\mathbb {F}}_q\) and both \(F_1=0\) and \(F_2=0\) are absolutely irreducible quadrics. Therefore, \(O_6(f_1,f_2)\) is not an ovoid.

If \(q=3^{2h}\) and \(a_1\notin \square _q\) or \(q=3^{2h+1}\) and \(a_1\in \square _q\) then both the factors \(F_1\) and \(F_2\) are not defined over \({\mathbb {F}}_q\) and both \(F_1=0\) and \(F_2=0\) are absolutely irreducible quadrics. Therefore the \({\mathbb {F}}_q\)-rational solutions satisfy \(F_1+F_2=0\) and \(F_1-F_2=0\), that is

$$\begin{aligned} {\left\{ \begin{array}{ll} E X_1 X_5 + 2EX_2 X_4 + 2X_3 + X_6&{}=0,\\ (EX_1 + \eta ^2 \sqrt{a_1} X_2 + 2E X_4 -\eta ^2\sqrt{a_1} X_5)(EX_1 -\eta ^2 \sqrt{a_1} X_2 + 2E X_4 +\eta ^2\sqrt{a_1} X_5)&{}=0.\\ \end{array}\right. } \end{aligned}$$

Since both the factors \(G_1\) and \(G_2\) of the second equation in the system above are not defined over \({\mathbb {F}}_q\), the \({\mathbb {F}}_q\)-rational solutions also satisfy \(G_1+G_2=0\) and \(G_1-G_2=0\), and thus

$$\begin{aligned} {\left\{ \begin{array}{ll} E X_1 X_5 + 2EX_2 X_4 + 2X_3 + X_6&{}=0,\\ X_1-X_4&{}=0,\\ X_2-X_5&{}=0.\\ \end{array}\right. } \end{aligned}$$

It is readily seen that the system above has only the solutions \(X_1-X_4=X_2-X_5=X_3-X_6=0\) and therefore \(O_6(f_1,f_2)\) is an ovoid. The claim follows by observing that the two conditions \(q=3^{2h}\) and \(a_1\notin \square _q\) or \(q=3^{2h+1}\) and \(a_1\in \square _q\) are equivalent to \(-a_1\) not being a square in \({\mathbb {F}}_q\). \(\square \)

2.3 Conclusion

Summing up, the only choice of \(f_1\) and \(f_2\) for which \(\textrm{O}_6(f_1,f_2)\) is an ovoid is

$$\begin{aligned} f_1(x,y,z)= & {} a_1 y^3+E^2x^2y-Exz;\\ f_2(x,y,z)= & {} \frac{E^4}{a_1}x^3+E^2y^2x+Eyz.\\ \end{aligned}$$

In this case the ovoid \(\textrm{O}_6(f_1,f_2)\) reads

$$\begin{aligned}{} & {} \Bigg \{\Bigg (1,x,y,z,a_1 y^3+E^2x^2y-Exz,\frac{E^4}{a_1}x^3+E^2y^2x+Eyz, -z^2-a_1 \left( y^2+\frac{E^2}{a_1}x^2\right) ^2\Bigg )\Bigg \}_{x,y,z\in \mathbb {F}_q}\\{} & {} \quad \cup \{(0,0,0,0,0,0,1)\}, \end{aligned}$$

which can be written as

$$\begin{aligned}{} & {} \Bigg \{\Bigg (1,x/E,y,z,a_1 y^3+x^2y-xz,E\left( x^3/a_1+y^2x+yz\right) , -z^2-a_1 \left( y^2+x^2/a_1\right) ^2\Bigg )\Bigg \}_{x,y,z\in \mathbb {F}_q}\\{} & {} \quad \cup \{(0,0,0,0,0,0,1)\}. \end{aligned}$$

For any \(E\in {\mathbb {F}}_q^*\) the above example is equivalent to Thas-Kantor via the projectivity

$$\begin{aligned} (X_0,X_1,X_2,X_3,X_4,X_5,X_6) \mapsto (X_0,E X_1,X_2,X_3,X_4,X_5/E,X_6) \end{aligned}$$

that fixes the quadric \(\textrm{Q}\).

Combining the previous propositions and with Theorem 5, we have proved the following

Theorem 21

Let \(f_1(X,Y,Z)=\sum _{ijk}a_{i,j,k}X^iY^jZ^k\) and \(f_2(X,Y,Z)=\sum _{ijk}b_{i,j,k}X^iY^jZ^k\). Suppose that \(q>6.3 (d+1)^{13/3}\), where \(\max \{\deg (f_1),\deg (f_2)\}=d\). If \(\textrm{O}_6(f_1,f_2)\) is an ovoid of Q(6, q) then

$$\begin{aligned} q=3^h, \quad f_1(X,Y,Z)=X^2Y-nY^3-XZ, \quad f_2(X,Y,Z)=-1/n X^3+XY^2+YZ. \end{aligned}$$

Hence \(O_6(f_1,f_2)\) is a Thas-Kantor ovoid.